LSAT and Law School Admissions Forum

Get expert LSAT preparation and law school admissions advice from PowerScore Test Preparation.

User avatar
 Dave Killoran
PowerScore Staff
  • PowerScore Staff
  • Posts: 5853
  • Joined: Mar 25, 2011
|
#47033
Complete Question Explanation
(The complete setup for this game can be found here: lsat/viewtopic.php?t=2214)

The correct answer choice is (D)

If Luis partners Sarah in dance 3, he must partner Rita in dance 2 and Tura in dance 1. This leaves either Rita or Sarah for Miguel to partner in dance 1, and thus answer choice (D) is correct.
 AlexaCrusinberry
  • Posts: 3
  • Joined: Jan 30, 2018
|
#43320
I want to understand why the respective answer was chosen on question 14.
Luis partners Sarah in dance 2

If Miguel partners Rita in dance 2.
Is it B because Karl partners Sarah in dance 1 or 2 we aren't sure.

Lets say Karl partners with Sarah in dance 1 then, technically Luis could partner with Sarah in 2. -- Is this the main reason why B is correct?

My question is what if we assumed Karl did partner with Sarah in dance 2.
Then Luis could not partner with Sarah in dance 2.

Do we assume the possibility that we can have Luis partner with Sarah in dance 2 because we do not know whether or not Karl is dancing with Sarah in dance 1 or 2?
 Emily Haney-Caron
PowerScore Staff
  • PowerScore Staff
  • Posts: 577
  • Joined: Jan 12, 2012
|
#43328
Hi Alexa,

Thanks for the question! You asked why Answer B is correct, but actually Answer D is the right answer here. I'll go ahead and walk you through the question.

The set-up for this game is extremely tricky, because there are some major inferences to make. Let's start there:
Rule 2 tells us that R's parter for 2 is S's partner for 3, which means that person has to be T's partner for 1
We can then infer that S's partner for 1 has to be T's partner for 2 (can't be S's again in 2, and can't be R's because she's partnered with whoever S is with for dance 3), which means that person is R's partner for dance 3
That means R's partner for 1 has to be S's partner for 2 and T's partner for 3.

So, if we have SL in dance 3 as provided by this question, we have RL for dance 2, and TL for dance 1
We don't know whether our SK block would be in dance 1 or dance 2, as you noted.
We now need to make sure the only other constraint we have (SK in either 1 or 2) doesn't come into play here. Following the inferences I wrote out above, If we have SK in 1, dance 1 would be SK, TL, and RM; dance 2 would be TK, RL, and then SM; dance 3 would be RK, SL, and then TM. That doesn't violate any rules. If we have SK in 2, dance 1 would be TL, SM, and RK; dance 2 would be SK, RL, and TM; and dance 3 would have to be SL, RM, and TK. That doesn't violate any rules, either.
So, with SL in dance three, there are two possible solutions to the game. In one, R is with M in dance 1; in the other, S is with M in dance 1. So, D is correct.

This one is hard, so let us know if you still have questions!

Get the most out of your LSAT Prep Plus subscription.

Analyze and track your performance with our Testing and Analytics Package.